Difference between revisions of "2024 AMC 10B Problems/Problem 5"

(Solution 1)
(Solution 1)
Line 10: Line 10:
 
==Solution 1==
 
==Solution 1==
  
Recall that the sum of the first <math>n</math> odd numbers is <math>n^2</math>. Thus <cmath>1 + 3 + 5 + 7\dots + 97 + 99 = 50^2 = 2500.</cmath>
+
Recall that the sum of the first <math>n</math> odd numbers is <math>n^2</math>. Thus <cmath>1 + 3 + 5 + 7+ \dots + 97 + 99 = 50^2 = 2500.</cmath>
  
 
If we want to minimize the number of sign flips to make the number negative, we must flip the signs corresponding to the largest absolute value. This will result in the inequality <cmath>1 + 3 + 5 +\dots + (2n - 3) + (2n - 1) - (2n + 1) - (2n + 3)-\dots - 97 - 99 < 0.</cmath>
 
If we want to minimize the number of sign flips to make the number negative, we must flip the signs corresponding to the largest absolute value. This will result in the inequality <cmath>1 + 3 + 5 +\dots + (2n - 3) + (2n - 1) - (2n + 1) - (2n + 3)-\dots - 97 - 99 < 0.</cmath>

Revision as of 04:04, 14 November 2024

The following problem is from both the 2024 AMC 10B #5 and 2024 AMC 12B #5, so both problems redirect to this page.

Problem

In the following expression, Melanie changed some of the plus signs to minus signs: \[1+3+5+7+...+97+99\] When the new expression was evaluated, it was negative. What is the least number of plus signs that Melanie could have changed to minus signs?

$\textbf{(A) } 14 \qquad\textbf{(B) } 15 \qquad\textbf{(C) } 16 \qquad\textbf{(D) } 17 \qquad\textbf{(E) } 18$

Solution 1

Recall that the sum of the first $n$ odd numbers is $n^2$. Thus \[1 + 3 + 5 + 7+ \dots + 97 + 99 = 50^2 = 2500.\]

If we want to minimize the number of sign flips to make the number negative, we must flip the signs corresponding to the largest absolute value. This will result in the inequality \[1 + 3 + 5 +\dots + (2n - 3) + (2n - 1) - (2n + 1) - (2n + 3)-\dots - 97 - 99 < 0.\]

The positive section of the sum will contribute $n^2$, and the negative section will contribute $-(2500-n^2) = (n^2 - 2500)$. The inequality simplifies to \[n^2 + (n^2 - 2500) < 0\] \[2n^2 < 2500\] \[n^2 < 1250\] The greatest positive value of $n$ satisfying the inequality is $n = 35$, corresponding to $35$ positive numbers, and $\boxed{\text{B. } 15}$ negatives.

~numerophile

See also

2024 AMC 10B (ProblemsAnswer KeyResources)
Preceded by
Problem 4
Followed by
Problem 6
1 2 3 4 5 6 7 8 9 10 11 12 13 14 15 16 17 18 19 20 21 22 23 24 25
All AMC 10 Problems and Solutions
2024 AMC 12B (ProblemsAnswer KeyResources)
Preceded by
Problem 4
Followed by
Problem 6
1 2 3 4 5 6 7 8 9 10 11 12 13 14 15 16 17 18 19 20 21 22 23 24 25
All AMC 12 Problems and Solutions

The problems on this page are copyrighted by the Mathematical Association of America's American Mathematics Competitions. AMC logo.png